Fourier transform of wave packet

Click For Summary
SUMMARY

The discussion focuses on the Fourier transform of a wave packet represented by the function ##g(x,t)=\frac{1}{2\pi}\int_{-\infty}^{\infty} a(k)e^{i(kx-\omega t)} \mathrm{d}k##. The participant aims to determine ##\hat{h}(k,t=0)## but struggles with the real part of the integral. The function ##a(k)## is defined as ##a(k)=A2\pi\delta(k)+2B/((k-C)^2+B^2)##, where ##A, B,## and ##C## are real constants, and ##\delta## denotes the Dirac delta function. The discussion highlights the complexities of extracting the real part from the integral.

PREREQUISITES
  • Understanding of Fourier transforms
  • Familiarity with wave packets in quantum mechanics
  • Knowledge of complex numbers and their properties
  • Experience with Dirac delta functions
NEXT STEPS
  • Study the properties of the Dirac delta function in Fourier analysis
  • Learn about the implications of real and imaginary parts in complex integrals
  • Explore advanced applications of Fourier transforms in quantum mechanics
  • Investigate numerical methods for evaluating Fourier transforms
USEFUL FOR

Students and professionals in physics, particularly those specializing in quantum mechanics, as well as mathematicians and engineers working with Fourier analysis and wave phenomena.

schniefen
Messages
177
Reaction score
4
Homework Statement
Consider ##h(x,t)=\frac{1}{2\pi}\int_{-\infty}^{\infty} \Re\{a(k)e^{i(kx-\omega t)}\}\mathrm{d}k.## What is the Fourier transform ##\hat{h}(k,t)## evaluated at ##t=0##, i.e. ##\hat{h}(k,t=0)##? (##a(k)## is given, but I do not think it needs to be specified)
Relevant Equations
The Fourier transform (FT) ##\hat{f}(k,t)=\int_{-\infty}^{\infty} f(x,t)e^{-ikx}\mathrm{d}x## and the inverse Fourier transform ##f(x,t)=\frac{1}{2\pi} \int_{-\infty}^{\infty} \hat{f}(k,t)e^{ikx}\mathrm{d}k##.
I am unsure if ##h(x,t)## really is a wave packet, but it looks like one, hence the title. Anyway, so I'd like to determine ##\hat{h}(k,t=0)##. My attempt so far is recognizing that, without the real part in the integral, i.e.

##g(x,t)=\frac{1}{2\pi}\int_{-\infty}^{\infty} a(k)e^{i(kx-\omega t)} \mathrm{d}k,##
then ##a(k)## is just the Fourier transform of ##g(x,t=0)##. However, I can not remove the real part from the integral and I am unsure how to proceed.
 
Physics news on Phys.org
schniefen said:
(##a(k)## is given, but I do not think it needs to be specified)
I disagree. What is a(k)?
 
DrClaude said:
I disagree. What is a(k)?
##a(k)=A2\pi\delta(k)+2B/((k-C)^2+B^2)##, where ##A, B## and ##C## are real constants. ##\delta## is the Dirac delta.
 
schniefen said:
However, I can not remove the real part from the integral and I am unsure how to proceed.

For complex numbers z and w, <br /> \Re(zw) = \Re(z)\Re(w) - \Im(z)\Im(w). However for w = e^{i\theta} = \cos \theta + i\sin \theta for real \theta you may prefer to write <br /> \begin{split}<br /> \Re(e^{i\theta}) &amp;= \frac{e^{i\theta} + e^{-i\theta}}2,\\<br /> \Im(e^{i\theta}) &amp;= \frac{e^{i\theta} - e^{-i\theta}}{2i}.\end{split}
 
Question: A clock's minute hand has length 4 and its hour hand has length 3. What is the distance between the tips at the moment when it is increasing most rapidly?(Putnam Exam Question) Answer: Making assumption that both the hands moves at constant angular velocities, the answer is ## \sqrt{7} .## But don't you think this assumption is somewhat doubtful and wrong?

Similar threads

Replies
1
Views
2K
  • · Replies 5 ·
Replies
5
Views
2K
  • · Replies 6 ·
Replies
6
Views
3K
  • · Replies 1 ·
Replies
1
Views
2K
  • · Replies 1 ·
Replies
1
Views
2K
  • · Replies 2 ·
Replies
2
Views
2K
  • · Replies 2 ·
Replies
2
Views
3K
Replies
5
Views
2K
  • · Replies 9 ·
Replies
9
Views
2K
  • · Replies 14 ·
Replies
14
Views
2K